20 POINTS! ***CORRECT*** ANSWER GETS BRAINLIEST!!!!
The fraction model below shows the steps that a student performed to find a quotient.
Which statement best interprets the quotient?
A. There are 5 1/6 three-fourths in 4 1/8
B. There are 5 1/6 three and one-eights in 3/4
C. There are 5 1/2 three and one-eights in 3/4
D. There are 5 1/2 three-fourths in 4 1/8

20 POINTS! ***CORRECT*** ANSWER GETS BRAINLIEST!!!! The Fraction Model Below Shows The Steps That A Student

Answers

Answer 1

Answer:

The answer is A pls mark me brainly


Related Questions

Kelsey had $65 to spend on books. Each book cost $5.50, and there was a $7.50 fee for shipping. She let b equal the number of books she can purchase and wrote the inequality 5.50 b + 7.5 less-than 65 to represent the situation. Which statements describe the reasoning used to determine if Kelsey’s inequality is correct? Select two options. The inequality symbol is correct because she must spend less than $65. The inequality symbol is incorrect because she can spend up to and including $65. The expression 5.50b + 7.5 is correct because $5.50 per book is 5.50b and that is added to the shipping fee of $7.50 to determine the total purchase price. The expression 5.50b + 7.5 is incorrect because $5.50 per book and $7.50 should be combined to $9.50b to determine the total purchase price. The inequality symbol is correct because she cannot spend more than $65.

Answers

The statements that can be used to describe the reasoning used to determine if Kelsey’s inequality is correct include:

The inequality symbol is incorrect because she can spend up to and including $65. The expression 5.50b + 7.5 is correct because $5.50 per book is 5.50b and that is added to the shipping fee of $7.50 to determine the total purchase price.

It should be noted that the inequality symbol is incorrect because she can spend up to and including $65.

Based on the information given, the correct expression that can be used to solve the question should be:

65 - (5.50b + 7.5)

In conclusion, the correct options are B and C.

Read related link on:

https://brainly.com/question/16904821

Answer:

B and C

Step-by-step explanation:

Tim owns a clothing store where he designs pairs of shorts, s, and T-shirts, t.
He sells the shorts for $12 and the T-shirts for $8 each. Tim can work 18
hours a day, at most. It takes him 30 minutes to design a T-shirt and 45
minutes to design a pair of shorts. He must design at least 12 items each
day, but he cannot design more than 30 items in one day. Which set of
inequalities below represents this scenario?
A. s 2 12 + $ ss 30 + tiss 24-0.66t, s2; 0; t2 0
B. s> 12-tss 30 - t Ss 24 -0.66t, s 2 0; t> 0
O c. s? 12-ts? 30-tss 24 -0.66t, s 2 0;t20
D. S 2 12-tss 30-ts 24 -0.66t, s 20; t 0​

Answers

Answer:

The correct option is;

B. s ≥ 12 - t, s ≤ 30 - t, s ≤ 24 - 0.66·t

Step-by-step explanation:

The given parameters are;

The number of T-shirts, t, and shorts, s, Tim must design a day = 12

The maximum number of T-shirts and shorts Tim can design a day = 30

The maximum number of hours Tim can work = 18 hours

Therefore, we have;

The number of shorts Tim designs in a day is ≥ The minimum number of T-shirts and shorts Tim must design a day less the number of T-shirts Tim designs

Which gives;

s ≥ 12 - t

Also the number of shorts Tim designs in a day is ≤ The maximum number of T-shirts, and shorts, Tim can design a day less the number of T-shirts Tim designs

Which gives;

s ≤ 30 - t

The number of 45 minute period for the design of shorts in 18 hours = 18×60/45 = 24

The fraction of 36 minutes in 45 minutes = 36/45 = 0.667

Therefore we have;

The number of shorts Tim designs in a day is ≤ The number of 45 minute periods in 18 hours less the number of 36 minutes periods used to design T-shirts

Which gives;

s ≤ 24 - 0.66·t

The correct option is s ≥ 12 - t, s ≤ 30 - t, s ≤ 24 - 0.66·t.

Answer:

B. s> 12-tss 30 - t Ss 24 -0.66t, s 2 0; t> 0

Step-by-step explanation:

Hope this helps!!

HELP ASAP
[tex]Given that $33^{-1} \equiv 77 \pmod{508}$, find $11^{-1} \pmod{508}$ as a residue modulo 508. (Give an answer between 0 and 507, inclusive.)[/tex]

Answers

Answer: 231

===================================================

Work Shown:

[tex]33^{-1} \equiv 77 \text{ (mod 508)}\\\\(3*11)^{-1} \equiv 77 \text{ (mod 508)}\\\\3^{-1}*11^{-1} \equiv 77 \text{ (mod 508)}\\\\3*3^{-1}*11^{-1} \equiv 3*77 \text{ (mod 508)}\\\\11^{-1} \equiv 231 \text{ (mod 508)}\\\\[/tex]

Notice how 33*77 = 2541 and 11*231 = 2541

[tex]2541 \equiv 1 \text{ (mod 508)}[/tex] since 2541/508 has a remainder of 1.

So effectively [tex]33*77 \equiv 1 \text{ (mod 508)}[/tex] and [tex]11*231 \equiv 1 \text{ (mod 508)}[/tex]

PLZ HELP !!!!!! ASAP!!!

Answers

Part (a)

BC = opposite side (furthest leg from the reference angle)

AB = adjacent side (closest leg from the reference angle)

AC = hypotenuse (always opposite the 90 degree angle)

=============================================

Part (b)

i. False. Angle B is 90 degrees as shown by the square angle marker.

ii. False. Side AB is opposite angle C. Note how "C" is part of "BC", so that means we cannot have BC be opposite C.

iii. True. Leg AB is the closer leg to angle A. We have "A" in "AB" to see this without having to draw the diagram. Refer to part (a) above.

iv. False. The longest side of any right triangle is always the hypotenuse. The longest side of any triangle is always opposite the largest angle.

==============================================

Part (c)

cos(theta) = adjacent/hypotenuse = AB/AC

tan(theta) = opposite/adjacent = BC/AB

Refer back to part (a) to determine the opposite,adjacent and hypotenuse side lengths.

==============================================

Part (d)

The reference angle has changed, so the opposite and adjacent sides swap. The hypotenuse remains the same regardless of what reference angle you pick.

sin(C) = opposite/hypotenuse = AB/AC

cos(C) = adjacent/hypotenuse = BC/AC

tan(C) = opposite/adjacent = AB/BC

Note the tangent ratio is the reciprocal of what we found back in part (c).

Answer & Step-by-step explanation:

(a)

The hypotenuse is on line CA (the hypotenuse is always opposite the 90° angle (marked by a little square))

The adjacent is on the line BA (adjacent is next to the given angle, but NOT the hypotenuse)

The opposite is on the line CB (this is opposite the given angle)

(b)

i.   false (b is a right angle)

ii.  false (the side opposite C is BA)

iii. true

iv. false (the side opposite B is the hypotenuse, and the hypotenuse is always the longest side in a triangle)

(c)

cosine ratio: [tex]cos=\frac{adjacent}{hypotenuse}[/tex]

tangent ratio: [tex]tan=\frac{opposite}{adjacent}[/tex]

The cosine and tangent ratios of the given angle:

[tex]cos0=\frac{AB}{CA} \\\\tan0=\frac{CB}{AB}[/tex]

(d)

Remember SOH-CAH-TOA:

Sine=Opposite/Hypotenuse

Cosine=Adjacent/Hypotenuse

Tangent=Opposite/Adjacent

Using the angle C, plug in the appropriate sides:

[tex]sinC=\frac{BA}{CA}\\\\ cosC=\frac{CB}{CA}\\\\ tanC=\frac{BA}{CB}[/tex]

:Done

hi there can you please help me

[tex]t = \sqrt{ \frac{ab - s}{r + ak} } [/tex]

Answers

[tex]t=\sqrt{\dfrac{ab-s}{r+ak}}\\\\t^2=\dfrac{ab-s}{r+ak}\\\\rt^2+akt^2=ab-s\\\\akt^2-ab=-rt^2-s\\\\a(kt^2-b)=-(rt^2+s)\\\\a=-\dfrac{rt^2+s}{kt^2-b}\\\\a=-\dfrac{rt^2+s}{-(b-kt^2)}\\\\a=\dfrac{rt^2+s}{b-kt^2}[/tex]

How many solutions does the nonlinear system of equations graphed below
have?
y
10+
-10
10
-10
A. One
B. Two
0
O
C. Four
O
D. Zero

Answers

Answer:

D. zero

Step-by-step explanation:

Since the graphs do not intersect, there are zero solutions.

The number of solutions on the graph is zero

How to determine the number of solutions?

The graph shows a linear equation (the straight line) and a non linear equation (the curve)

From the graph, we can see that the straight line and the curve do not intersect

This means that the graph do not have any solution

Hence, the number of solutions on the graph is zero

Read more about non-linear graphs at:

https://brainly.com/question/16274644

#SPJ5

Which statement correctly compares
1–201 and
1512
ol-201 = 151
ol-201 < 51
l-201 > 151

Answers

Answer:

Option B.

Step-by-step explanation:

Consider the correct question is "Which statement correctly compares

1. -201  and  151

-201 = 151

-201 < 51

-201 > 151"

The given numbers are -201 and 151. We need to compare these numbers.

We know that all negative numbers are less than positive numbers.

So,

-201 < 151

If both numbers are negative, then the larger negative number is the smaller number.

Therefore, the correct option is B.

Shaquira is baking cookies to put in packages for a fundraiser. Shaquira has made 86 8686 chocolate chip cookies and 42 4242 sugar cookies. Shaquira wants to create identical packages of cookies to sell, and she must use all of the cookies. What is the greatest number of identical packages that Shaquira can make?

Answers

Answer: 2

Step-by-step explanation:

Given: Shaquira has made 86  chocolate chip cookies and 42 sugar cookies.

Shaquira wants to create identical packages of cookies to sell, and she must use all of the cookies.

Now, the greatest number of identical packages that Shaquira can make= GCD of 86 and 42

Prime factorization of 86 and 42:

86 = 2 ×43

42 = 2 × 3 × 7

GCD of 86 and 42 = 2   [GCD = greatest common factor]

Hence, the greatest number of identical packages that Shaquira can make =2

The generic version was basedOn the brand name and was 2/3 the size of the brand name. If the generic television set is 12 inches by 24 inches what are the dimensions of the brand name television

Answers

Answer:

18 inches by 36 inches.

Step-by-step explanation:

Since we have given that

The generic version was basedOn the brand name and was 2/3

And given Dimensions of generic version is given by 12inches ×24inches

If we use the first dimensions of 12inches we have

12=2/3 × brand

12×3/2 = brand

=18inches= brand

we use the first dimensions of 24 inches we have

24=2/3 × brand

24×3/2 = brand

=36 inches= brand

brand= 36 inches

Therefore,the dimensions of brand name will be 18 inches by 36 inches.

Solve: 5x2 + 25x = 0

Answers

Answer:

x = -0.4

x = -(2/5)

Answer:

x = ± √5

Step-by-step explanation:

Please indicate exponentiation by using the symbol " ^ ":

5x^2 + 25x = 0

Divide all three terms by 5.  We get:

x^2 + 5 = 0, or x^2 = -5

Then x = ± √5

Ramona works in a clothing store where she earns a base salary of $140 per day plus 14% of her daily sales. She sold $600 in clothing on Saturday and $1200 in clothing on Sunday. How much did she earn over the two days? A. $252 B. $291 C. $392 D. $532

Answers

Answer:

I hope this helps!

Answer D

Step-by-step explanation:

Step-by-step explanation:

salary per day =$140

bonus on sales =14%

sales on Saturday =$600

bonus on Saturday sales=14/100*$600

=$84

sales on Sunday =$1200

bonus on Sunday sales=14/100*$1200

=$168

total amount she earned over the two days=$140+$84+$168

=$532

Question 1 (
Multiple Choice Worth 3 points)
(07.04)
The cost of 3 slices of pizza is $4.89. What is the cost of each slice of pizza?
O $1.63
$1.89
O $2.45
O $2.88​

Answers

Answer:

Each slice of pizza cost:

$1.63

Step-by-step explanation:

4.89/3 = 1.63

Answer:

$1.63

Step-by-step explanation:

We want to find the cost per slice of pizza. Therefore, we must divide the total cost by the number of slices of pizza.

cost / slices

It costs $4.89 for 3 slices.

$4.89 / 3 slices

Divide 4.89 by 3 (4.89/3=1.63)

$1.63 / slice

The cost of each slice of pizza is $1.63

what is the value of this expression when g= -3.5?
8-|2g-5|
a. 20
b. 10
c. 6
d. -4​

Answers

Answer:

d. -4

Step-by-step explanation:

Let's plug in g

8 - |2(-3.5) - 5|

8 - |-7-5|

8 - |-12|

The absolute value is always positive of any number,

8 - 12

= -4

Answer:

D. -4

Step-by-step explanation:

We are given this expression:

[tex]8-|2g-5|[/tex]

and asked to evaluate when g= -3.5 Therefore, we must substitute -3.5 in for g.

[tex]8-|2(-3.5)-5|[/tex]

First, multiply 2 and -3.5

2 * -3.5 = -7

[tex]8-|-7-5|[/tex]

Next, subtract 5 from -7.

-7-5= -12

[tex]8-|-12|[/tex]

Next, evaluate the absolute value of -12. The absolute value is how far away a number is from 0, and it is always positive. The absolute value of -12 is 12.

[tex]8-12[/tex]

Subtract 12 from 8.

[tex]-4[/tex]

The value of the expression is -4 and D is the correct answer.

Write and solve an equation to answer the question. How many people must attend the third show so that the average attendance per show is 3000?

Answers

Answer:

3250

Step-by-step explanation:

so for the first and 2nd show, the attendance is 2580 and 2920.

The average of both these numbers is 2750

the if the third show had 3000 people, the average attendance would only be 2875.

We need the average number to be 3000.

2750 is 250 less than 3000, so the other number must be 250 more.

3250 is how many people should go to the last show.

Answers:First box = (5500+x)/3Second box = 3500

=====================================

Explanation:

We have 2580 people attend the first show and 2920 attend the second. So far, that's 2580+2920 = 5500 people. Add on another x people to get 5500+x, which represents the sum of all three days attendance figures. Divide this sum by 3 to get the average attendance

average attendance = (sum of individual attendance values)/(number of days)

average attendance = (5500+x)/3

So that's why (5500+x)/3 goes in the first box. The parenthesis are important to ensure that you divide all of "5500+x" over 3. If you just wrote 5500+x/3, then the computer would think you just want to divide x only over 3.

----------------

We set (5500+x)/3 equal to 3000 as we want the average of the three days to be 3000

(5500+x)/3 = 3000

5500+x = 3*3000

5500+x = 9000

x = 9000-5500

x = 3500

We need 3500 people to show up on day 3 so that the average of all three days is 3000.

3500 goes in the second box.

----------------

Check:

The figures for the three days are 2580, 2920, and 3500

They add to 2580+2920+3500 = 9000

Which divides to 9000/3 = 3000, which is the average we're after. So the answer is confirmed.

I need Helpppp quick!!!!

Answers

Answer:

G

Step-by-step explanation:

let his fixed price be x and his hourly fee be y;

270 = 4y + x

420 = 7y + x

x is common in both equations

equate the two;

x = 270-4y and x = 420-7y

270-4y = 420-7y

3y = 150

y = 50

x = 270-4*50

x = 70

This table represents a quadratic function.
y
x
0
14
1
10.5
2
8
3
6.5
4
5
6.5
What is the value of a in the function's equation?
A.2
B.1/2
C.-1/2
D.1

Answers

Answer:

B. 1/2

Step-by-step explanation:

y = ax^2 + bx + c

14 = a(0)^2 + b(0) + c

c = 14

10.5 = a(1)^2 + b(1) + 14

10.5 = a + b + 14 ____(i)

8 = a(2)^2 + b(2) + 14

8 = 4a + 2b + 14

4 = 2a + b + 7 ___ (ii)

i - ii

10.5 - 4 = -a + 7

6.5 = -a + 7

a = 7- 6.5

a = 0.5

Value of a in the quadratic function is 0.5

What is Quadratic function?

In algebra, a quadratic function, a quadratic polynomial, a polynomial of degree 2, or simply a quadratic, is a polynomial function with one or more variables in which the highest-degree term is of the second degree

Given,

Quadratic function

y = [tex]ax^{2}+bx+c[/tex]

Consider values in the table x= 0 and  y =14

[tex]14=a(0)^{2}+b(0)+c\\ c=14[/tex]

Consider x=1 and y = 10.5

[tex]10.5=a(1^{2})+b(1)+c\\ a+b=10.5-14\\a+b=-3.5[/tex]

Consider x=2 and y =8

[tex]8=a(2^{2})+b(2)+c\\ a\\8=4a+2b+14\\4a+2b=-6\\2a+b=-3[/tex]

Subtract a + b= -3.5 from 2a + b= -3

a=-3--3.5=0.5

Hence, the Value of a in the quadratic function is 0.5

Learn more about Quadratic function here

https://brainly.com/question/5975436

#SPJ2

Find the amplitude of y = -2 sin x

Answers

Answer:

Amplitude = 2

Step-by-step explanation:

The amplitude of this sine wave is 2 denoted by the coefficient -2 in front of the sin(x).  The negative of the coefficient denotes that the sine wave is the opposite of the standard sine wave.

Cheers.

how do you solve 2m-10=44+8m

Answers

Answer:

m = -9

Step-by-step explanation:

2m-10=44+8m

Subtract 2m from each side

2m-2m-10=44+8m-2m

-10 = 44+6m

Subtract 44 from each side

-10-44 = 44-44+6m

-54 = 6m

Divide by 6

-54/6 = 6m/6

-9 = m

Answer:

solve by solving the salvation for equation don't be a slave get educated from what's gave

Samantha’s college runs on a trimester schedule so she receives a bill 3 times a year for tuition. Each trimester costs $1,450, and Samantha must complete 2 years of college to receive her degree. The average cost for books each trimester is $350. Approximately what will be the total cost for Samantha to get her degree?

Answers

Answer:

10800

Step-by-step explanation:

1 trimesters cost = 1450 + 350  $

2 year -> 6 trimester

1800$ x 6 = 10800 $

PLEASE HELP
Find the area and the perimeter of the shaded regions below. Give your answer as a completely simplified exact value in terms of π (no approximations). The figures below are based on semicircles or quarter circles and problems b), c), and d) are involving portions of a square.

Answers

Answer:

perimeter is  4 sqrt(29) + 4pi  cm

area is 40 + 8pi cm^2

Step-by-step explanation:

We have a semicircle and a triangle

First the semicircle with diameter 8

A = 1/2 pi r^2 for a semicircle

r = d/2 = 8/2 =4

A = 1/2 pi ( 4)^2

  =1/2 pi *16

  = 8pi

Now the triangle with base 8 and height 10

A = 1/2 bh

  =1/2 8*10

  = 40

Add the areas together

A = 40 + 8pi cm^2

Now the perimeter

We have 1/2 of the circumference

1/2 C =1/2 pi *d

         = 1/2 pi 8

        = 4pi

Now we need to find the length of the hypotenuse of the right triangles

using the pythagorean theorem

a^2+b^2 = c^2

The base is 4 ( 1/2 of the diameter) and the height is 10

4^2 + 10 ^2 = c^2

16 + 100 = c^2

116 = c^2

sqrt(116) = c

2 sqrt(29) = c

Each hypotenuse is the same so we have

hypotenuse + hypotenuse + 1/2 circumference

2 sqrt(29) + 2 sqrt(29) + 4 pi

4 sqrt(29) + 4pi  cm

Step-by-step explanation:

First we need to deal with the half circle. The radius of this circle is 4, because the diameter is 8. The formula for the circumference of a circle is 2piR.

2pi4 so the perimeter for the half circle would be 8pi/2.

The area of that half circle would be piR^2 so 16pi/2.

Now moving on the triangle part, we need to find the hypotenuse side of AC. We will use the pythagoram theorem. 4^2+10^2=C^2

16+100=C^2

116=C^2

C=sqrt(116)

making the perimeter of this triangle 2×sqrt(116)

The area of this triangle is 8×10=80, than divided by 2 which is equal to 40.

We than just need to add up the perimeters and areas for both the half circle and triangle.

The area would be equal to 8pi+40

The perimeter would be equal to 4pi+4(sqrt(29))

Estimate. Then determine the area. Please please please, need help!

Answers

Estimate:

2.3 rounds down to 2

So after multiplying by 2, the area is estimated to be 4 cm squared.

Actual Area:

2.3 x 2 = 4.6

The actual area of the shape is 4.6 cm squared.

Hope this helped!

Answer:

4.6

Step-by-step explanation:

Represents the solution to the inequality -9=2/3x-7<5

Answers

Answer:

-3=x <13

Step-by-step explanation:

[tex] - 9 = \frac{2x}{3} - 7 < 5[/tex]

Multiply through by 3

[tex] - 27 = 2x - 21 < 15[/tex]

Add 21 to all sides

[tex] - 6 = 2x < 36[/tex]

Divide through by 2

[tex] - 3 = x < 18[/tex]

The solutin set is

[tex]{- 3 = x < 18}[/tex]

Please answer this question now

Answers

Answer:

11 yd

Step-by-step explanation:

To find the volume of a rectangular prism, we multiply the width, length and height.

We already know the length, 18, and the height, 11, and the volume, 2178, so we can easily solve for y.

[tex]18\cdot y\cdot11=2178\\192y=2178\\y = 11[/tex]

Hope this helped!

I need helps will give you a good rating.

Answers

Answer: x = 3

Step-by-step explanation:

Sqrt(x+7) - 1  = x

Sqrt(x+7) = x + 1

x+7 = x^2 + 1

x = x^2 - 6

x=3

What is the smallest positive integer $n$ such that $\frac{n}{n+101}$ is equal to a terminating decimal?

Answers

Answer:

n = 24

Step-by-step explanation:

Given the fraction:

[tex]$\frac{n}{n+101}$[/tex]

To find:

Smallest positive integer [tex]$n$[/tex] such that the fraction is equal to a terminating decimal.

Solution:

The rule that a fraction is equal to a terminating decimal states that, the denominator must contain factors of only 2 and 5.

i.e. Denominator must look like [tex]2^m\times 5^n[/tex], only then the fraction will be equal to a terminating decimal.

Now, let us have a look at the denominator, [tex]n+101[/tex]

Let us use hit and trial method to find the value of [tex]n[/tex] as positive integer.

n = 1, denominator becomes 102 = [tex]2 \times 3 \times 17[/tex] not of the form [tex]2^m\times 5^n[/tex].

n = 4, denominator becomes 105 = [tex]5 \times 3 \times 7[/tex] not of the form [tex]2^m\times 5^n[/tex].

n = 9, denominator becomes 110 = [tex]2 \times 5 \times 11[/tex] not of the form [tex]2^m\times 5^n[/tex].

n = 14, denominator becomes 115 = [tex]5 \times 23[/tex] not of the form [tex]2^m\times 5^n[/tex].

n = 19, denominator becomes 120 = [tex]5 \times 3 \times 2^3[/tex] not of the form [tex]2^m\times 5^n[/tex].

n = 24, denominator becomes 125 = [tex]2^0 \times 5 ^3[/tex] It is of the form [tex]2^m\times 5^n[/tex].

So, the answer is n = 24

1. Suzette ran and biked for a total of 80 miles in 9 hours. Her average running speed was 5 miles per hour (mph) and her average biking speed was 12 mph. Let x = total hours Suzette ran. Let y = total hours Suzette biked. Use substitution to solve for x and y. Show your work. Check your solution. (a) How many hours did Suzette run? (b) How many hours did she bike?

Answers

Answer:

a) Suzette ran for 4 hours

b) Suzette biked for 5 hours

Step-by-step explanation:

Speed is rate of distance traveled, it is the ratio of distance traveled to time taken. It is given by:

Speed = distance / time

The total distance ran and biked by Suzette (d) = 80 miles, while the total time ran and biked by Suzette (t) = 9 hours.

For running:

Her speed was 5 miles per hour, let the total hours Suzette ran be x and the total distance she ran be p, hence since Speed = distance / time, therefore:

5 = p / x

p = 5x

For biking:

Her speed was 12 miles per hour, let the total hours Suzette ran be y and the total distance she ran be q, hence since Speed = distance / time, therefore:

12 = q / y

q = 12y

The total distance ran and biked by Suzette (d) = Distance biked + distance ran

d = p + q

80 = p + q

80 = 5x + 12y                 (1)

The total time taken to run and bike by Suzette (t) = time spent to bike + time spent to run

t = x + y

9 = x + y                         (2)

Solving equation 1 and equation 2, multiply equation 2 by 5 and subtract from equation 1:

7y = 35

y = 35/7

y = 5 hours

Put y = 5 in equation 2:

9 = x + 5

x = 9 -5

x = 4 hours

a) Suzette ran for 4 hours

b) Suzette biked for 5 hours

NEED ASAP What is the quotient and remainder of 8,595 ÷ 24?

Answers

Answer:

358.125

Step-by-step explanation:

Answer:

358 3/24

Step-by-step explanation:  

Find the value of x so that the function has the given value.


j(x)=−4/5x+7; j(x)=−5

x=



Answers

Answer:

x = 3

Step-by-step explanation:

j(x) = 4/5(-5) + 7

= -4 + 7

= 3

Answer:

15

Step-by-step explanation: -4/5 x has to be -12 because -12+7 equals 5. Since we want to figure out x, we have to flip -4/5 x to 4/5x which would change the -12 to 12. What is a fourth of 12? It is three. 12+3 equals 15. This is the first right answer on all of the internet for this question!

Consider the function represented by 9x + 3y = 12 with x as the independent variable. How can this function be
written using function notation?
O FID = - Šv
O f(x) = - 3x + 4
Of(x) = -x +
O fly) = -34+4​

Answers

Answer:

f(x) = - 3x + 4

Step-by-step explanation:

Note that y = f(x)

Rearrange making y the subject

9x + 3y = 12 ( subtract 9x from both sides )

3y = - 9x + 12 ( divide all terms by 3 )

y = - 3x + 4 , that is

f(x) = - 3x + 4

simplify 5 x 5^2 in index form

Answers

Answer:

5x(25)

Step-by-step explanation:

Other Questions
Which of these could be an option after high school for a person who wants to further his education but doesn't want to take out student loans? Select the correct answer. In which area is composing for games similar to composing for movies? A. royalties earned B. success of song-based soundtracks C. usage of audio software for mixing and editing D. spotting sessions In Macroland autonomous consumption equals 100, the marginal propensity to consume equals 0.75, net taxes are fixed at 40, planned investment is fixed at 50, government purchases are fixed at 150, and net exports are fixed at 20. Planned aggregate expenditure equals:________a.1,000. b.1,160. c.1,280. d.1,440. 5/14, 7/10, 5/6, 11/15, 19/2 An unstable particle at rest spontaneously breaks into two fragments of unequal mass. The mass of the first fragment is 3.00 10-28 kg, and that of the other is 1.86 10-27 kg. If the lighter fragment has a speed of 0.844c after the breakup, what is the speed of the heavier fragment What is the area of the house (including the drawing room, TV room, balcony, hallway, kitchen, and bedroom)? A line runs tangent to a circle at the point (4, 2). The line runs through the origin. Find the slope of the tangent line. NEED HELP 200 POINT IF YOU RIGHT AND BRAILY Best Buy is currently selling the latest model of the iPadPro for $549.99. Since you are an employee there, youreceive a 5% discount. How much will the iPad Pro costyou if you use your employee discount (before taxes). The Securities and Exchange Commission requires companies listing on the New York Stock Exchange and the Nasdaq Stock Market to have codes of ethics. A code of ethics is What is the main purpose of state legislatures?A. to provide legislative support for CongressB. to challenge the legality of federal lawsC. to control the legislation of local governmentsD. to make and pass laws at the state levelE. to provide financial support for the federal government If Superman really had x-ray vision at 0.12 nm wavelength and a 4.1 mm pupil diameter, at what maximum altitude could he distinguish villains from heroes, assuming that he needs to resolve points separated by 5.4 cm to do this? -5+2(10b-2)=31 need help thanks! A charity organization is holding a food drive with a goal to collect at least 1,000 cans offood by the end of the month. It currently has 565 cans from donations and is having anevent where 87 guests will attend and bring cans. Which solution set represents thenumber of cans each guest must bring to meet the goal?+OA++012345678910---+OB. 4+012345678910OC.+1235678910OD. ++++-6+7.+0123458910 Help me please please please please help please, got a little lost in everything This painting by Correggio, Assumption of the Virgin, is an example of the art technique which involves the graduation of light and shadow to produce a hazy effect. What is the term used for this technique? Magine that you work for an advertising agency, and you've been tasked with developing an advertising campaign to increase sales of COLA. How would you develop an advertisement for this product that uses a Human Psychology Factor. * g An increase in taxes when the economy is above full employment ______ aggregate demand and real GDP, and the price level ______. "Literacy is prerequisite of development. Justify this statement with examples.